tea.mathoverflow.net - Discussion Feed (Is this question suitable for MO?) Sun, 04 Nov 2018 13:48:51 -0800 http://mathoverflow.tqft.net/ Lussumo Vanilla 1.1.9 & Feed Publisher Ryan Budney comments on "Is this question suitable for MO?" (17394) http://mathoverflow.tqft.net/discussion/1236/is-this-question-suitable-for-mo/?Focus=17394#Comment_17394 http://mathoverflow.tqft.net/discussion/1236/is-this-question-suitable-for-mo/?Focus=17394#Comment_17394 Sat, 03 Dec 2011 12:44:43 -0800 Ryan Budney The author did not word the question very well -- depending on specifics the answer could be "obviously no" or "yes with caveats". It strikes me as potentially an intro manifolds homework problem. I've voted to close.

]]>
deane.yang comments on "Is this question suitable for MO?" (17381) http://mathoverflow.tqft.net/discussion/1236/is-this-question-suitable-for-mo/?Focus=17381#Comment_17381 http://mathoverflow.tqft.net/discussion/1236/is-this-question-suitable-for-mo/?Focus=17381#Comment_17381 Sat, 03 Dec 2011 07:35:20 -0800 deane.yang
It seems to me that this can be answered directly from definitions and maybe some elementary lemmas. Does this really belong in math.stackexchange.com? ]]>